Contracts Questions

Lakukan tugas rumah & ujian kamu dengan baik sekarang menggunakan Quizwiz!

Ozzie owned and occupied Blackacre, which was a tract of land improved with a one-family house. His friend, Victor, orally offered Ozzie $50,000 for Blackacre, the fair market value, and Ozzie accepted. Because they were friends, they saw no need for attorneys or written contracts and shook hands on the deal. Victor paid Ozzie $5,000 down in cash and agreed to pay the balance of $45,000 at an agreed closing time and place. Before the closing, Victor inherited another home and asked Ozzie to return his $5,000. Ozzie refused, and, at the time set for the closing, Ozzie tendered a good deed to Victor and declared his intention to vacate Blackacre the next day. Ozzie demanded that Victor complete the purchase. Victor refused. The fair market value of Blackacre has remained $50,000. In an appropriate action brought by Ozzie against Victor for specific performance, if Ozzie loses, the most likely reason will be that (A) The agreement was oral. (B) Keeping the $5,000 is Ozzie's exclusive remedy. (C) Victor had a valid reason for not closing. (D) Ozzie remained in possession on the day set for the closing.

(A) The agreement was oral.

Laura resolved to pull a prank on Billy. She revealed to her friends that she would pretend to offer Billy $300 for his old laptop computer. She knew that Billy needed money and that his computer had a market value of just $50. When Laura offered Billy $300 for his computer, she explained her interest by saying, "I think there's a collector's item underneath all those dents and scuffs." Billy blushed but immediately replied, "I accept your offer and will deliver the computer to you tomorrow." Laura said, "Thanks," and walked away. When Billy presented Laura with the computer the next day, Laura and her assembled friends burst into laughter. "You doofus!" Laura said, "Nobody wants your junky laptop! I was just pulling a prank and you fell for it." Billy stormed off. Based on these facts, it is most likely that: (A) A contract exists because Billy commenced performance. (B) A contract exists because Billy reasonably believed that Laura made a serious offer, which he accepted. (C) No contract exists because Laura did not intend to make an offer. (D) No contract exists because Laura told her friends that the offer was made in jest.

(B) A contract exists because Billy reasonably believed that Laura made a serious offer, which he accepted.

Under the terms of a written contract, Karp agreed to construct for Manor a garage for $10,000. Nothing was said in the parties' negotiations or in the contract about progress payments during the course of the work. After completing 25% of the garage strictly according to Manor's specifications, Karp demanded payment of $2,000 as a "reasonable progress payment." Manor refused, and Karp abandoned the job. If each party sues the other for breach of contract, which of the following will the court decide? A) Both parties are in breach, and each is entitled to damages, if any, from the other. (B) Only Karp is in breach and liable for Manor's damages, if any. (C) Only Manor is in breach and liable for Karp's damages, if any. (D) Both parties took reasonable positions and neither is in breach.

(B) Only Karp is in breach and liable for Manor's damages, if any.

Five years ago, Sally acquired Blackacre, improved with a 15-year-old dwelling. This year Sally listed Blackacre for sale with Bill, a licensed real estate broker. Sally informed Bill of several defects in the house that were not readily discoverable by a reasonable inspection, including a leaky basement, an inadequate water supply, and a roof that leaked. Paul responded to Bill's advertisement, was taken by Bill to view Blackacre, and decided to buy it. Bill saw to it that the contract specified the property to be "as is" but neither Bill nor Sally pointed out the defects to Paul, who did not ask about the condition of the dwelling. After closing and taking possession, Paul discovered the defects, had them repaired, and demanded that Sally reimburse him for the cost of the repairs. Sally refused and Paul brought an appropriate action against Sally for damages. If Sally wins, it will be because (A) Sally fulfilled the duty to disclose defects by disclosure to Bill. (B) The contract's "as is" provision controls the rights of the parties. (C) Bill became the agent of both Paul and Sally and thus knowledge of the defects was imputed to Paul. (D) The seller of a used dwelling that has been viewed by the buyer has no responsibility toward the buyer.

(B) The contract's "as is" provision controls the rights of the parties.

A homeowner and a builder entered into a written contract to add a spare room on to the homeowner's home at a cost of $13,000. The contract contained a clause stating that the builder will not begin construction without prior approval of the plans by the homeowner's financial advisor. The builder submitted his designs to both the homeowner and the financial advisor. The homeowner liked the plans, but the financial advisor did not and withheld his approval. The builder asked the homeowner whether she wanted him to submit new designs. The homeowner told the builder orally, "No! Your designs are great, so go right ahead and construct the new room." The builder constructed the new room. The homeowner now refuses to pay the builder, citing the clause requiring approval by the financial advisor. If the builder sues the homeowner, the builder will recover: (A) The full contract price, because the financial advisor's approval was not a condition precedent for the contract to take effect. (B) The full contract price, because the builder detrimentally relied on the homeowner's waiver of the condition precedent. (C) The reasonable value of the builder's services and materials, because otherwise the homeowner would be unjustly enriched. (D) Nothing, because the homeowner's oral statement will be excluded by the parol evidence rule.

(B) The full contract price, because the builder detrimentally relied on the homeowner's waiver of the condition precedent.

Dominique obtained a bid of $10,000 to tear down her old building and another bid of $90,000 to replace it with a new structure in which she planned to operate a sporting goods store. Having only limited cash available, Dominque asked Hardcash for a $100,000 loan. After reviewing the plans for the project, Hardcash in a signed writing promised to lend Dominique $100,000 secured by a mortgage on the property and repayable over ten years in equal monthly installments at 10% annual interest. Dominique promptly accepted the demolition bid and the old building was removed, but Hardcash thereafter refused to make the loan. Despite diligent efforts, Dominique was unable to obtain a loan from any other source. Does Dominique have a cause of action against Hardcash? (A) Yes, because by having the building demolished, she accepted Hardcash's offer to make the loan. (B) Yes, because her reliance on Hardcash's promise was substantial, reasonable, and foreseeable. (C) No, because there was no bargained-for exchange of consideration for Hardcash's promise to make the loan. (D) No, because Dominique's inability to obtain a loan from any other source demonstrated that the project lacked the financial soundness that was a constructive condition to Hardcash's performance.

(B) Yes, because her reliance on Hardcash's promise was substantial, reasonable, and foreseeable.

Ohner and Planner signed a detailed writing in which Planner, a landscape architect, agreed to landscape and replant Ohner's residential property in accordance with a design prepared by Planner and incorporated in the writing. Ohner agreed to pay $10,000 for the work upon its completion. Ohner's spouse was not a party to the agreement, and had no ownership interest in the premises. Shortly before the agreement was signed, Ohner and Planner orally agreed that the writing would not become binding on either party unless Ohner's spouse should approve the landscaping design. If Ohner's spouse disapproves the design and Ohner refuses to allow Planner to proceed with the work, is evidence of the oral agreement admissible in Planner's action against Ohner for breach of contract? (A) Yes, because the oral agreement required approval by a third party. (B) Yes, because the evidence shows that the writing was intended to take effect only if the approval occurred. (C) No, because the parol evidence rule bars evidence of a prior oral agreement even if the latter is consistent with the terms of a partial integration. (D) No, because the prior oral agreement contradicted the writing by making the parties' duties conditional.

(B) Yes, because the evidence shows that the writing was intended to take effect only if the approval occurred.

Pater and his adult daughter, Carmen, encountered Tracey, an old family friend, on the street. Carmen said to Tracey, "How about lending me $1,000 to buy a used car? I'll pay you back with interest one year from today." Pater added, "And if she doesn't pay it back as promised, I will." Tracey thereupon wrote out and handed to Carmen his personal check, payable to her, for $1,000, and Carmen subsequently used the funds to buy a used car. When the debt became due, both Carmen and Pater refused to repay it, or any part of it. In an action by Tracey against Pater to recover $1,000 plus interest, which of the following statements would summarize Pater's best defense? (A) He received no consideration for his conditional promise to Tracey. (B) His conditional promise to Tracey was not to be performed in less than a year from the time it was made. (C) His conditional promise to Tracey was not made for the primary purpose of benefiting himself (Pater). (D) The loan by Tracey was made without any agreement concerning the applicable interest rate.

(C) His conditional promise to Tracey was not made for the primary purpose of benefiting himself (Pater).

Ohner and Planner signed a detailed writing in which Planner, a landscape architect, agreed to landscape and replant Ohner's residential property in accordance with a design prepared by Planner and incorporated in the writing. Ohner agreed to pay $10,000 for the work upon its completion. At Ohner's insistence, the written Ohner-Planner agreement contained a provision that neither party would be bound unless Ohner's law partner, an avid student of landscaping, should approve Planner's design. Before Planner commenced the work, Ohner's law partner, in the presence of both Ohner and Planner, expressly disapproved the landscaping design. Nevertheless, Ohner ordered Planner to proceed with the work, and Planner reluctantly did so. When Planner's performance was 40% complete, Ohner repudiated his duty, if any, to pay the contract price or any part thereof. If Planner now sues Ohner for damages for breach of contract, which of the following concepts best supports Planner's claim? (A) Substantial performance. (B) Promissory estoppel. (C) Irrevocable waiver of condition. (D) Unjust enrichment.

(C) Irrevocable waiver of condition.

On August 1, Geriatrics, Inc., operating a "lifetime care" home for the elderly, admitted Ohlster, who was 84 years old, for a trial period of two months. On September 25, Ohlster and Geriatrics entered into a written lifetime care contract with an effective commencement date of October 1. The full contract price was $20,000, which, as required by the terms of the contract, Ohlster prepaid to Geriatrics on September 25. Ohlster died of a heart attack on October 2. In a restitutionary action, can the administrator of Ohlster's estate, a surviving sister, recover on behalf of the estate either all or part of the $20,000 paid to Geriatrics on September 25? (A) Yes, because Geriatrics would otherwise be unjustly enriched at Ohlster's expense. (B) Yes, under the doctrine of frustration of purpose. (C) No, because Ohlster's life span and the duration of Geriatrics' commitment to him was a risk assumed by both parties. (D) No, but only if Geriatrics can show that between September 25 and Ohlster's death it rejected, because of its commitment to Ohlster, an application for lifetime care from another elderly person.

(C) No, because Ohlster's life span and the duration of Geriatrics' commitment to him was a risk assumed by both parties.

Facts from prior question: Dominique obtained a bid of $10,000 to tear down her old building and another bid of $90,000 to replace it with a new structure in which she planned to operate a sporting goods store. Having only limited cash available, Dominque asked Hardcash for a $100,000 loan. After reviewing the plans for the project, Hardcash in a signed writing promised to lend Dominique $100,000 secured by a mortgage on the property and repayable over ten years in equal monthly installments at 10% annual interest. Dominique promptly accepted the demolition bid and the old building was removed, but Hardcash thereafter refused to make the loan. Despite diligent efforts, Dominique was unable to obtain a loan from any other source. Assume the same facts as the last question. For this question only, assume that Dominique has a cause of action against Hardcash. If she sues him for monetary relief, what is the probable measure of her recovery? (A) Expectancy damages, measured by the difference between the value of the new building and the old building, less the amount of the proposed loan ($100,000). (B) Expectancy damages, measured by the estimated profits from operating the proposed sporting goods store for ten years, less the cost of repaying a $100,000 loan at 10% interest over ten years. (C) Reliance damages, measured by the $10,000 expense of removing the old building, adjusted by the decrease or increase in the market value of Dominique's land immediately thereafter. (D) Nominal damages only, because both expectancy and reliance damages are speculative, and there is no legal or equitable basis for awarding restitution.

(C) Reliance damages, measured by the $10,000 expense of removing the old building, adjusted by the decrease or increase in the market value of Dominique's land immediately thereafter.

Ames had painted Bell's house under a contract which called for payment of $2,000. Bell, contending in good faith that the porch had not been painted properly, refused to pay anything. On June 15, Ames mailed a letter to Bell stating, "I am in serious need of money. Please send the $2,000 to me before July 1." On June 18, Bell replied, "I will settle for $1,800 provided that you agree to repaint the porch." Ames did not reply to this letter. Thereafter, Bell mailed a check for $1,800 marked "Payment in full on the Ames-Bell painting contract as per letter dated June 18." Ames received the check on June 30. Because he was badly in need of money, Ames cashed the check without objection and spent the proceeds but has refused to repaint the porch. Bell's refusal to pay anything to Ames when he finished painting was a (A) partial breach of contract only if Ames had properly or substantially painted the porch. (B) partial breach of contract whether or not Ames had properly or substantially painted the porch. (C) total breach of contract only if Ames had properly or substantially painted the porch. (D) total breach of contract whether or not Ames had properly or substantially painted the porch.

(C) total breach of contract only if Ames had properly or substantially painted the porch.

Same facts as the prior question. Which of the following, if proved, would best support Mural's defense? (A) Gennybelle gave Mural no consideration for an irrevocable sub-bid. (B) Mural's sub-bid expressly requested Gennybelle's acceptance after awarding of the main contract. (C) Even after paying $18,000 for the paperhanging, Gennybelle would make a net profit of $100,000 on the Doctors' Building contract. (D) Before submitting her own bid, Gennybelle had reason to suspect that Mural had made a computational mistake in figuring his sub-bid.

(D) Before submitting her own bid, Gennybelle had reason to suspect that Mural had made a computational mistake in figuring his sub-bid.

Albert engaged Bertha, an inexperienced actress, to do a small role in a new Broadway play for a period of six months at a salary of $800 a week. Bertha turned down another role in order to accept this engagement. On the third day of the run, Bertha was hospitalized with influenza and Helen was hired to do the part. A week later, Bertha recovered, but Albert refused to accept her services for the remainder of the contract period. Bertha then brought an action against Albert for breach of contract. Which of the following is Bertha's best legal theory? (A) Her acting contract with Albert was legally severable into weekly units. (B) Her performance of the literal terms of the contract was physically impossible. (C) Her reliance on the engagement with Albert by declining another acting role created an estoppel against Albert. (D) Her failure to perform for one week was not a material failure so as to discharge Albert's duty to perform.

(D) Her failure to perform for one week was not a material failure so as to discharge Albert's duty to perform.

A company contracted with a builder to construct a new corporate headquarters for a fixed price of $100 million. At the time of the contract, structural steel was widely available and was included in the contract as a $6 million item. Before work began on the project, tornado damage shut down the production facility of the biggest structural steel supplier in the country, and the price of structural steel increased by 20% as a result. The builder informed the company of the steel price increase, and the parties then orally agreed to increase the project price to $101 million. The builder proceeded with construction and delivered the project on time. The company paid the builder $100 million but refused to pay the additional $1 million. If the builder sues the company for $1 million, is the builder likely to prevail? (A) No, because the modification was never reduced to a writing signed by the party to be charged. (B) No, because there was no consideration for the modification of the contract. (C) Yes, because the company's promise was supported by consideration. (D) Yes, because the modification was fair and equitable in view of the unanticipated increase in the price of structural steel.

(D) Yes, because the modification was fair and equitable in view of the unanticipated increase in the price of structural steel.

On February 3, a property owner mailed an offer to a married couple who had expressed an interest in buying his property at 337 Green Street, consisting of a house and lot. The offer asked for $200,000, "terms $60,000 cash, with the balance secured by a first mortgage." The offer reached the couple on February 5. On February 8, the couple replied by fax that the offer had been received and was being considered, and added, "We would much prefer a straight cash deal. Would you consider an immediate purchase for $180,000 cash?" On February 10, this reply was received by the property owner, who responded with a one-word fax: "No." Receiving this fax on February 11, the couple faxed: "Fax received. We accept your offer of February 3." If the property owner now refuses to sell and the couple sues, the court would probably hold: A) A valid contract exists. B) No contract exists because the couple's response of February 8 operated to terminate the property owner's offer. C) No contract exists because the couple's communications to the property owner contained material alterations of the terms of the offer. D) No contract exists because the offer relates to real property, and the communications fail to establish the terms of the proposed agreement with sufficient definiteness.

A) A valid contract exists.

An elderly woman regularly corresponded with her only niece (her sister's daughter), who lived out of town. One day she sent her niece a letter telling her that she planned to leave everything she owned to her upon her death. When the woman died, her will left her entire estate valued at $200,000 to her nephew (her niece's only brother). The nephew wrote his sister a letter telling her that he felt bad about being the only person named in their aunt's will, and added, "I'm going to share her estate with you. We can discuss the details at Auntie's funeral." The niece spent $800 on round-trip tickets to attend her aunt's funeral. After the funeral, she spoke with her brother, who told her that he had changed his mind about sharing their aunt's estate with her. He went on to state that he would be willing to share the estate with her if she were willing to share their mother's estate with him, when their mother passed on. The niece responded by telling him that their mother had already signed over all her (the mother's) property to her. He replied that, given her attitude, he would keep their aunt's estate for himself. Later, after the two had returned to their respective homes, no longer on speaking terms, the niece sued the nephew for a 50% share of their aunt's estate. What amount should the niece realize from her suit? A) Nothing, because the aunt's will left everything to the nephew, and the nephew's letter is an insufficient basis to compel him to share. B) $800, because this represents the niece's actual expenses incurred in reliance on the nephew's letter. C) $100,000 (half of the aunt's estate), because the nephew promised her that in the letter. D) $100,000, but only if she shares their mother's estate with the nephew.

A) Nothing, because the aunt's will left everything to the nephew, and the nephew's letter is an insufficient basis to compel him to share.

Seisin and Vendee, standing on Greenacre, orally agreed to its sale and purchase for $5,000, and orally marked its bounds as "that line of trees down there, the ditch that intersects them, the fence on the other side, and that street on the fourth side." In which of the following is the remedy of reformation most appropriate? A. As later reduced to writing, the agreement by clerical mistake included two acres that are actually beyond the fence. B. Vendee reasonably thought that two acres beyond the fence were included in the oral agreement but Seisin did not. As later reduced to writing, the agreement included the two acres. C. Vendee reasonably thought that the price orally agreed upon was $4,500, but Seisin did not. As later reduced to writing, the agreement said $5,000. D. Vendee reasonably thought that a dilapidated shed backed up against the fence was to be torn down and removed as part of the agreement, but Seisin did not. As later reduced to writing, the agreement said nothing about the shed.

A. As later reduced to writing, the agreement by clerical mistake included two acres that are actually beyond the fence.

On April 1, Owner and Buyer signed a writing in which Owner, "in consideration of $100 to be paid to Owner by Buyer," offered Buyer the right to purchase Greenacre for $100,000 within 30 days. The writing further provided, "This offer will become effective as an option only if and when the $100 consideration is in fact paid." On April 20, Owner, having received no payment or other communication from Buyer, sold and conveyed Greenacre to Citizen for $120,000. On April 21, Owner received a letter from Buyer enclosing a cashier's check for $100 payable to Owner and stating, "I am hereby exercising my option to purchase Greenacre and am prepared to close whenever you're ready." Which of the following, if proved, best supports Buyer's suit against Owner for breach of contract? A. Buyer was unaware of the sale to Citizen when Owner received the letter and check from Buyer on April 21. (Seller should have let buyer know he was selling it to someone else) B. On April 15, Buyer decided to purchase Greenacre, and applied for and obtained a commitment from Bank for a $75,000 loan to help finance the purchase. C. When the April 1 writing was signed, Owner said to Buyer, "Don't worry about the $100; the recital of '$100 to be paid' makes this deal binding." D. Owner and Buyer are both professional dealers in real estate.

A. Buyer was unaware of the sale to Citizen when Owner received the letter and check from Buyer on April 21. (Seller should have let buyer know he was selling it to someone else)

A dairy farmer hired a local company to assemble milking machines that the farmer had purchased. The written contract between the parties provides that the company would assemble and install the milking machines in the farmer's dairy barn within 30 days, in time for the arrival of additional cows, and the farmer agreed to pay the company $10,000. Three weeks into the job, the company realized that it would lose $2,500 on the job, due to a new wage agreement forced on the company by its employees' union after the contract was executed. The company approached the farmer and told him that the job could not be completed for less than $12,500. After some discussion, the farmer and the company executed an agreement obligating the farmer to pay an additional $2,500 upon completion of the job. The company completed the work on time, but the farmer now refuses to pay the additional $2,500. In a suit by the company against the farmer, which of the following would be the farmer's strongest position? A. He has no duty to pay the company more than $10,000, because this was a contract for services and the modification was not supported by consideration. B. The modification is voidable because the company knew that the farmer needed the machines up and running in 30 days and took advantage of his duress. C. The company's mistake regarding the cost of providing its services is not grounds for voiding the original contract. D. During initial contract negotiations, the company assured the farmer that the milking machines would be assembled and installed for no more than $10,000.

A. He has no duty to pay the company more than $10,000, because this was a contract for services and the modification was not supported by consideration.

On April 10, the owner of a small farm mailed a letter to a new resident of the area who had expressed an interest in buying the farm. In his letter, the farm owner offered to sell the farm to the resident for $100,000. The offer expressly stated that the offer expires on June 1, "if acceptance by the offeree has not been received by the offeror on or before that date." On the morning of June 1, the resident sent a written acceptance to the farm owner by messenger. However, through negligence of the messenger company, the acceptance was not delivered to the farm owner until June 2. On June 4, the owner entered into a contract to sell the farm to another buyer for more money but did not inform the resident of the transaction. When the resident followed up by phone on June 10, the farm owner told him that he had sold the farm to another buyer. Which of the following is the most correct statement? A. No contract between the farm owner and the resident arose on June 2. B. An enforceable contract arose on June 1. C. The farm owner's silence constituted an acceptance of the resident's message on June 2. D. A voidable contract arose on June 1.

A. No contract between the farm owner and the resident arose on June 2.

On July 18, Snowco, a shovel manufacturer, received an order for the purchase of 500 shovels from Acme, Inc., a wholesaler. Acme had mailed the purchase order on July 15. The order required shipment of the shovels no earlier than September 15 and no later than October 15. Typed conspicuously across the front of the order form was the following: "Acme, Inc. reserves the right to cancel this order at any time before September 1." Snowco's mailed response, saying, "We accept your order," was received by Acme on July 21. As of July 22, which of the following is an accurate statement as to whether a contract was formed? A. No contract was formed because of Acme's reservation of the right to cancel. B. No contract was formed because Acme's order was only a revocable offer. C. A contract was formed, but prior to September 1 it was terminable at the will of either party. D. A contract was formed, but prior to September 1 it was an option contract terminable at will of Acme.

A. No contract was formed because of Acme's reservation of the right to cancel.

Becky, age 92, has only $25,000 to her name and is fearful that she will outlive her financial resources. On August 10, 2012, Becky explains her concern to her wealthy grand-nephew Logan. On that same day, by signed writing, Logan makes her this promise: "When you exhaust the 25,000 you now have, I will provide you, as a gift, any amount of money you request, up to a maximum of $50,000 per year for the remainder of your life." In September 2012, Becky's grandson Thane asks Becky for $25,000. Unable to resist, and believing that she can turn to Logan for any money she may need, Becky gives Thane the $25,000 in her bank account—all that she has in the world. Becky then contacts Logan. Explaining what she has done, she asks him for $25,000. Logan responds, "I did not make my promise to you so that you could give the money away to Thane. I'm not going to keep the promise." To what extent does the doctrine of promissory estoppel require that Logan keep his promise of August 10, 2012? A. Not at all, because Becky did not reasonably rely on it. B. To the extent of $25,000, because that is the extent to which Becky relied on it. C. To the extent of $25,000, because that is less than amount he promised to pay per year. D. Fully, because Becky was reasonable in believing that Logan would honor his promise.

A. Not at all, because Becky did not reasonably rely on it.

Rob, an auto retailer, had an adult daughter, Betsy, who needed a car in her employment but had only $3,000 with which to buy one. Rob wrote to her, "Give me $3,000 and I'll give you the car on the lot that we have been using as a demonstrator." Betsy thanked her father and paid him the $3,000. As both Rob and Betsy knew, the demonstrator was reasonably worth $10,000. After Betsy had paid the $3,000, but before the car had been delivered to her, one of Rob's sales staff sold and delivered the same car to a customer for $10,000. Neither the salesperson nor the customer was aware of the transaction between Rob and Betsy. Does Betsy have an action for breach of contract? A. Yes, because Rob's promise was supported by bargained-for consideration. B. Yes, because Rob's promise was supported by the moral obligation a father owes his child as to the necessities of life. C. No, because the payment of $3,000 was inadequate consideration to support Rob's promise. D. No, because the salesperson's delivery of the car to the customer revoked Rob's offer.

A. Yes, because Rob's promise was supported by bargained-for consideration.

On Dec. 15, Lawyer received from Stationer, Inc., a retailer of office supplies, an offer consisting of its catalog and a signed letter stating, "We will supply you with as many of the items in the enclosed catalog as you order during the next calendar year. We assure you that this offer and the prices in the catalog will remain firm throughout the coming year." Assume that no other correspondence passed between Stationer and Lawyer until the following April 15 (4 months later), when Stationer received from Lawyer a faxed order for "100 reams of your paper, catalog item # 101." Did Lawyer's April 15 fax constitute an effective acceptance of Stationer's offer at the prices specified in the catalog? A. Yes, because Stationer had not revoked its offer before April 15. B. Yes, because a one-year option contract had been created by Stationer's offer. C. No, because under applicable law the irrevocability of Stationer's offer was limited to a period of three months. D. No, because Lawyer did not accept Stationer's offer within a reasonable time.

A. Yes, because Stationer had not revoked its offer before April 15.

A consumer buys a candy bar (Almond Joy) from a grocery store, bites into it, and breaks a tooth on a piece of metal imbedded in the candy bar. Does the consumer have a claim for breach of warranty against the store? A. Yes, because a candy bar that breaks a tooth is probably not merchantable. B. No, because the consumer assumed the risk of such injury. C. No, because if anyone is to blame, it is the manufacturer and not the seller. D. No, because no promises were made about the candy bar.

A. Yes, because a candy bar that breaks a tooth is probably not merchantable.

John and Mary agreed on the telephone that John would buy Mary's Ted Williams autographed baseball for $400. Mary later told him, "The joke's on you. I got a better offer for the ball, and all I had with you was an oral contract, and that doesn't count." Is there an enforceable contract between Mary and John? A. Yes, because oral agreements for the sale of goods for less than $500 are enforceable. B. Yes, because the UCC does not apply to this agreement because John and Mary are not merchants. C. Yes, because Mary admitted making the agreement. D. No, because there is no writing signed by Mary.

A. Yes, because oral agreements for the sale of goods for less than $500 are enforceable.

Brenda and Sarah are law students. Brenda tells Sarah (a senior editor on the law review) that she needs word processing software to write her law review article. Sarah says, "I just bought new software, so I'll sell you my old software." Brenda buys the software and discovers that although it otherwise works perfectly well, it does not create footnotes and is therefore not useful for writing her law review article. Assuming Article 2 applies to this transaction, has Sarah breach the implied warranty of fitness for a particular purpose? A. Yes, because she impliedly promised that the software would create footnotes. B. Yes, because the software is not fit for the ordinary purpose of word processing. C. No, because neither party is a merchant. D. No, because no warranty was given in writing.

A. Yes, because she impliedly promised that the software would create footnotes.

A corporation whose subsidiaries include a major hotel chain planned to build a new hotel and advertised for bids to build the hotel within the next six months. Four bids were received, for $17 million, $17.2 million, $17.4 million, and $15 million. The corporation's chief financial officer reviewed the bids, then emphatically told the corporation's chief executive officer ("CEO") that there was "no way" the low bidder could make a profit on the $15 million bid. The CEO made no response. In fact, the builder had stayed up for 72 hours without sleep preparing the bid for the hotel project and had neglected to include the plumbing expenses in the bid. Typically, the cost of plumbing, including the shop's profit, would have been about $2 million. Shortly after the $15 million contract was signed by the CEO and the builder, the builder discovered his mistake and telephoned the CEO to tell her that he had forgotten to include the cost of plumbing, adding that he would normally charge $2 million for plumbing. The CEO agreed to pay the additional $2 million, but this arrangement was never reduced to writing. After the builder completed the project on time, the CEO sent him a check for only $15 million. Can the builder compel the CEO to tender the additional $2 million? A. Yes, because the CEO was on notice of the builder's mistake. B. Yes, because the builder relied to his detriment on the CEO's promise. C. No, because the builder had a preexisting legal duty to complete the project for $15 million. D. No, because evidence of the agreement to pay the additional $2 million is barred by the Statute of Frauds.

A. Yes, because the CEO was on notice of the builder's mistake.

A written construction contract began with the following recital: "This Agreement, between Land, Inc. (hereafter called 'Owner'), and Builder, Inc., and Boss, its President (hereafter called 'Contractor'), witnesseth:" The signatures to the contract appeared in the following format: LAND, INC. By /s/ Oscar Land President BUILDER, INC. By /s/ George Mason Vice President /s/ Mary Boss, President Mary Boss Builder, Inc., became insolvent and defaulted. Land, Inc. sued Boss individually for the breach, and at the trial Boss proffered evidence from the pre-contract negotiations that only Builder, Inc. was to be legally responsible for performing the contract. If the court finds the contract to be completely integrated, is Boss's proffered evidence admissible? A. Yes, because the writing is ambiguous as to whether or not Boss was intended individually to be a contracting party. B. Yes, because the evidence would contradict neither the recital nor the form of Boss's signature. C. No, because the legal effect of Boss's signature cannot be altered by evidence of prior understandings. D. No, because of the application of the "four corners" rule, under which the meaning of a completely integrated contract must be ascertained solely from its own terms.

A. Yes, because the writing is ambiguous as to whether or not Boss was intended individually to be a contracting party.

In month eight, Restaurant sells its business to Conglomerate, which owns a chain of restaurants. As part of the sale, Restaurant assigns its rights in the Bakery contract to Conglomerate. Conglomerate determines that $5 per pie is a good price, so it orders 600 pies in month nine, most of which will be sold in Conglomerate's restaurants in other states. Is Bakery obligated to supply 600 pies?

Agreement is reassignable but must use terms from original deal, so unreasonably disproportionate.

A contractor sends a company its bid on a construction job. The bid consists of specifications and prices, followed by a number of boilerplate provisions. The company responds by writing up the price and specifications on its own form, which it sends to the contractor. The contractor completes the project and the company inspects it after 25 days. The company finds some unfinished work and demands payment for it. The contractor claims that he is not responsible for the unfinished work because the form he sent says that inspection must be completed within 20 days after construction, and the company did not timely inspect. However, the company's form indicates that it has 30 days after completion to inspect. Who is responsible for the unfinished work? A. The owner, because the contractor's form governs. B. The contractor, because the company's form governs. C. Neither, because there is no agreement on this term. D. Both terms are knocked out and the party who would be responsible is supplied by trade usage or by a court.

B. The contractor, because the company's form governs.

A builder learned of a bid being let by the local school board for a new high school. Anxious to get the job, the builder immediately advertised for sub-bids. The lowest sub-bid came from a local contractor. It was for $180,000, which was $10,000 less than the next lowest sub-bid. The builder submitted its general bid to the school board after computing it based, in part, on the lowest sub-bid. The builder's general bid was $10,000 lower than the school board had originally anticipated. Pleasantly surprised by the low bid, the school board accepted. After the acceptance, the contractor who had submitted the lowest sub-bid informed the builder that there had been a mistake in computing the sub-bid. The correct sub-bid was $200,000. The builder had been unaware of this mistake. In a suit for rescission of its contract with the builder, the contractor should: A) Not prevail, because the mistake in the figures was made before the bid was accepted. B) Not prevail, because nothing indicates that the builder actually knew or should have known of the mistake involved. C) Prevail, because the mistake was part of the basis of the bargain. D) Prevail, because this was a unilateral, not a bilateral, mistake.

B) Not prevail, because nothing indicates that the builder actually knew or should have known of the mistake involved.

On May 1, Seller, a computer wholesaler, orally agrees to sell 50 IBM laptop computers to Buyer, a computer retailer, for $1,000 per computer to be delivered within 30 days and paid for upon delivery. On May 2, Buyer sends the following hand-written note to Seller (which Seller received): • Bought 50 IBM laptops from Seller. /s/Buyer Seller does nothing for the next 15 days. On May 16, Seller telephones Buyer and tells Buyer that the deal is off. Buyer brings suit against Seller for breach of contract. Seller asserts the Statute of Frauds.

Binding under confirmation

Same facts as No. 40, except Buyer's writing stated: Bought 25 IBM laptops from Seller. /s/Buyer

Binding under confirmation for 25.

Same facts as No. 40, except Buyer, not Seller, breached the oral agreement. Seller sues Buyer.

Binding under confirmation for 25.

During negotiations between two friends over the sale of a boat, the seller tells the buyer that the engine was warranted to be free of defects for a period of two years. The parties finally sign a written agreement that contains no warranty terms. After 14 months, the engine develops problems because of a defect. A court finds that the writing is a partial integration and that the promise was actually made. Is the seller liable under the warranty? A) Yes, because the gap fillers will supply an implied warranty. B) Yes, because the oral promise does not contradict the terms of the writing. C) No, because the writing was a final expression of their agreement as to the warranty. D) No, because the buyer would have a claim even under the written warranty.

B) Yes, because the oral promise does not contradict the terms of the writing.

Seller and Buyer enter into an agreement by which Seller agrees to sell her 1964 Ford Mustang to Buyer for $50,000. Both parties believe the car is worth $50,000. Two months after the sale, Buyer discovers that the car is worth only $10,000. As a result, Buyer brings suit to rescind the contract. What is the likely outcome of such litigation? A. Buyer will prevail because Buyer mistakenly believed the car was worth $50,000. B. Buyer will prevail because Seller mistakenly believed car was worth $50,000. C. Buyer will prevail because both Buyer and Seller mistakenly believed the car was worth $50,000. D. Seller will prevail.

D. Seller will prevail.

On May 1, Seller, a computer wholesaler, orally agrees to sell 50 IBM laptop computers to Buyer, a computer retailer, for $1,000 per computer to be delivered within 30 days and paid for upon delivery. On May 16, Seller telephones Buyer and tells Buyer that the deal is off. Buyer brings suit against Seller for breach of contract. In her answer, Seller admits that the parties had an oral agreement for 25 laptops at $1,000 apiece, but denies the remainder. Seller also asserts the Statute of Frauds.

Enforceable for 25 because seller admitted to the deal for the 25.

On May 1, Seller, a computer wholesaler, orally agrees to sell 50 IBM laptop computers to Buyer, a computer retailer, for $1,000 per computer to be delivered within 30 days. Buyer pays (and Seller accepts) a $20,000 down payment on May 1. On May 16, Seller telephones Buyer and tells Buyer that the deal is off. Buyer brings suit against Seller for breach of contract. Seller asserts the Statute of Frauds. What is the result?

Enough to satisfy SOF for 20 computers (what he paid for), but not for all of them.

Buyer faxed the following signed message to seller, his longtime widget supplier: "Urgently need blue widgets. Ship immediately three gross at your current list price of $600." Upon receipt of the fax, seller shipped three gross of red widgets to buyer and faxed to buyer the following message: "Temporarily out of blue. In case red will help, I am shipping three gross at the same price. Hope you can use them." Upon buyer's timely receipt of both the shipment and seller's fax, which of the following best describes the rights and duties of buyer and seller? A. Buyer may accept the shipment, in which case he must pay seller the list price, or he must reject the shipment and recover from seller for total breach of contract. B. Buyer may accept the shipment, in which case he must pay seller the list price, or he may reject the shipment, in which case he has no further rights against seller. C. Buyer may accept the shipment, in which case he must pay seller the list price, less any damages sustained because of the nonconforming shipment, or he may reject the shipment and recover from seller for total breach of contract, subject to seller's right to cure. D. Buyer may accept the shipment, in which case he must pay seller the list price, less any damages sustained because of the nonconforming shipment, or he may reject the shipment provided that he promptly covers by obtaining conforming widgets from another supplier.

B. Buyer may accept the shipment, in which case he must pay seller the list price, or he may reject the shipment, in which case he has no further rights against seller.

Fixtures, Inc., in a signed writing, contracted with Apartments for the sale to Apartments of 50 identical sets of specified bathroom fixtures, 25 sets to be delivered on March 1, and the remaining 25 sets on April 1. The agreement did not specify the place of delivery, or the time or place of payment. Which of the following statements is correct? A. Fixtures must tender 25 sets to Apartments at Apartments place of business on March 1, but does not have to turn them over to Apartments until Apartments pays the contract price for the 25 sets. B. Fixtures has no duty to deliver the 25 sets on March 1 at Fixtures place of business unless Apartments tenders the contract price for the 25 sets on that date. C. Fixtures must deliver 25 sets on March 1, and Apartments must pay the contract price for the 25 sets within a reasonable time after their delivery. D. Fixtures must deliver 25 sets on March 1, but Apartments payment is due only upon the delivery of all 50 sets.

B. Fixtures has no duty to deliver the 25 sets on March 1 at Fixtures place of business unless Apartments tenders the contract price for the 25 sets on that date.

Mural, a wallpaper hanger, sent Gennybelle, a general contractor, this telegram: "Will do all paperhanging on new Doctors' Building, per owner's specs, for $14,000 if you accept within reasonable time after main contract awarded." /s/ Mural. Three other competing hangers sent Gennybelle similar bids in the respective amounts of $18,000, $19,000, and $20,000. Gennybelle used Mural's $14,000 figure in preparing and submitting her own sealed bid on Doctors' Building. Before the bids were opened, Mural truthfully advised Gennybelle that the former's telegraphic sub-bid had been based on a $4,000 computational error and was therefore revoked. Shortly thereafter, Gennybelle was awarded the Doctors' Building construction contract and subsequently contracted with another paperhanger for a price of $18,000. Gennybelle now sues Mural to recover $4,000. Which of the following, if proved, would most strengthen Gennybelle's prospect of recovery? A. After Mural's notice of revocation, Gennybelle made a reasonable effort to subcontract with another paperhanger at the lowest possible price. B. Gennybelle had been required by the owner to submit a bid bond and could not have withdrawn or amended her bid on the main contract without forfeiting that bond. C. Mural was negligent in erroneously calculating the amount of his sub-bid. D. Gennybelle dealt with all of her subcontractors in good faith and without seeking to renegotiate (lower) the prices they had bid.

B. Gennybelle had been required by the owner to submit a bid bond and could not have withdrawn or amended her bid on the main contract without forfeiting that bond.

In a writing signed by both parties on December 1, K agreed to buy from S a gasoline engine for $1,000, delivery to be made on the following February 1. Through a secretarial error, the writing called for delivery on March 1, but neither party noticed the error until February 1. Before signing the agreement, K and S orally agreed that the contract of sale would be effective only if K should notify S in writing no later than January 2 that K had arranged to resell the engine to a third party. Otherwise, they orally agreed, "There is no deal." On December 15, K entered into a contract with T to resell the engine to T at a profit. K did not give S notice of the resale until January 25, and S received it by mail on January 26. Meantime, the value of the engine had unexpectedly increased about 75% since December 1, and S renounced the agreement. If K sues S on February 2 for breach of contract, which of the following is S's best defense? A. The secretarial error in the written delivery-term was a mutual mistake concerning a basic fact, and the agreement is voidable by either party. B. K's not giving written notice by January 2 of his resale was a failure of a condition precedent to the existence of a contract. C. In view of the unexpected 75% increase in value of the engine after December 1, S's performance is excused by the doctrine of commercial frustration. D. The agreement, if any, is unenforceable because a material term is not included in the writing.

B. K's not giving written notice by January 2 of his resale was a failure of a condition precedent to the existence of a contract.

Retailer, a dry goods retailer, telephoned Manufacturer, a towel manufacturer, and offered to buy for $5 each a minimum of 500 and a maximum of 1,000 large bath towels, to be delivered in 30 days. Manufacturer orally accepted this offer and promptly sent the following letter to Retailer, which Retailer received two days later: "This confirms our agreement today by telephone to sell you 500 large bath towels for 30-day delivery. /s/ Manufacturer." Twenty-eight days later, Manufacturer tendered to Retailer 1,000 (not 500) conforming bath towels, all of which Retailer rejected because it had found a better price term from another supplier. Because of a glut in the towel market, Manufacturer cannot resell the towels except at a loss. In a suit by Manufacturer against Retailer, which of the following will be the probable decision? A. Manufacturer can enforce a contract for 1,000 towels, because Retailer ordered and Manufacturer tendered that quantity. B. Manufacturer can enforce a contract for 500 towels, because Manufacturer's letter of confirmation stated that quantity term. C. there is no enforceable agreement, because Retailer never signed a writing. D. there is no enforceable agreement, because Manufacturer's letter of confirmation did not state a price term.

B. Manufacturer can enforce a contract for 500 towels, because Manufacturer's letter of confirmation stated that quantity term.

Prior question: On Dec. 15, Lawyer received from Stationer, Inc., a retailer of office supplies, an offer consisting of its catalog and a signed letter stating, "We will supply you with as many of the items in the enclosed catalog as you order during the next calendar year. We assure you that this offer and the prices in the catalog will remain firm throughout the coming year." Assume that no other correspondence passed between Stationer and Lawyer until the following April 15 (4 months later), when Stationer received from Lawyer a faxed order for "100 reams of your paper, catalog item # 101." Did Lawyer's April 15 fax constitute an effective acceptance of Stationer's offer at the prices specified in the catalog? For this question only, assume that on Jan. 15, having at that time received no reply from Lawyer, Stationer notified Lawyer that effective Feb. 1, it was increasing the prices of certain items in its catalog. Is the price increase effective with respect to catalog orders Stationer receives from Lawyer during the month of February? A. No, because Stationer's original offer, including the price term, became irrevocable under the doctrine of promissory estoppel. B. No, because Stationer is a merchant with respect to office supplies, and its original offer, including the price term, was irrevocable throughout the month of February. C. Yes, because Stationer received no consideration to support its assurance that it would not increase prices. D. Yes, because the period for which Stationer gave assurance that it would not raise prices was longer than three months.

B. No, because Stationer is a merchant with respect to office supplies, and its original offer, including the price term, was irrevocable throughout the month of February.

Tenant rented a commercial building from Landlord, and operated a business in it. The building's large front window was smashed by vandals six months before expiration of the Tenant-Landlord lease. Tenant, who was obligated thereunder to effect and pay for repairs in such cases, promptly contracted with Glazier to replace the window for $2,000, due 30 days after satisfactory completion of the work. Sixty days later, Tenant mailed a $1,000 check to Glazier bearing on its face the following conspicuous notation: "This check is in full and final satisfaction of your $2,000 window replacement bill." Without noticing this notation, Glazier cashed the check and now sues Tenant for the $1,000 difference. If Tenant's only defense is accord and satisfaction, is Tenant likely to prevail? A. No, because Glazier failed to notice Tenant's notation on the check. B. No, because the amount owed by Tenant to Glazier was due and undisputed. C. Yes, because by cashing the check Glazier impliedly agreed to accept the $1,000 as full payment of its claim. D. Yes, because Glazier failed to write a reservation-of-rights notation on the check before cashing it.

B. No, because the amount owed by Tenant to Glazier was due and undisputed.

Client consulted Lawyer about handling the sale of Client's building, and asked Lawyer what her legal fee would be. Lawyer replied that her usual charges was $100 per hour, and estimated that the legal work on behalf of Client would cost about $5,000 at that rate. Client said, "Okay; let's proceed with it," and Lawyer timely and successfully completed the work. Because of unexpected title problems, Lawyer reasonably spent 75 hours on the matter and shortly thereafter mailed Client a bill for $7,500, with a letter itemizing the work performed and time spent. Client responded by a letter expressing his good-faith belief that Lawyer had agreed to a total fee of no more than $5,000. Client enclosed a check in the amount of $5,000 payable to Lawyer and conspicuously marked, "Payment in full for legal services in connection with the sale of Client's building." Despite reading the "Payment in full..." language, Lawyer, without any notation of protest or reservation of rights, endorsed and deposited the check to her bank account. The check was duly paid by the Client's bank. A few days later, Lawyer unsuccessfully demanded payment from the Client of the $2,500 difference between the amount of her bill and the check, and now sues Client for that difference. What, if anything, can Lawyer recover from Client? A. Nothing, because the risk of unexpected title problems in a real-property transaction is properly allocable to the seller's attorney and thus to Lawyer in this case. B. Nothing, because the amount of Lawyer's fee was disputed in good faith by Client, and Lawyer impliedly agreed to an accord and satisfaction. C. $2,500, because Client agreed to an hourly rate for as many hours as the work reasonably required, and the sum of $5,000 was merely an estimate. D. The reasonable value of Lawyer's services in excess of $5,000, if any, because there was no specific agreement on the total amount of Lawyer's fee.

B. Nothing, because the amount of Lawyer's fee was disputed in good faith by Client, and Lawyer impliedly agreed to an accord and satisfaction.

A general contractor about to bid on a construction job with an owner invited a carpenter to bid on the carpentry work along with several others. The carpenter agreed to bid if the contractor would agree to give the carpenter the job provided that the carpenter's bid was lowest and the contractor was awarded the main contract. The contractor so agreed. The carpenter, expending time and money in preparing his bid, submitted the lowest carpentry bid of $100,000. The contractor used the carpenter's bid in calculating his own bid, which was successful. Which of the following best supports the carpenter's position that the contractor is obligated to award the carpentry subcontract to the carpenter? A. The carpenter incurred an economic detriment in preparing his bid. B. The carpenter gave consideration for the contractor's conditional promise to award the carpentry subcontract to him. C. The contractor has an obligation to the owner to subcontract with the carpenter because the carpenter's bid was used in calculating the contractor's bid, and the carpenter is a third-party intended beneficiary of that obligation. D. The contractor has an implied duty to deal fairly and in good faith with all bidders whose bids the contractor used in calculating his main bid.

B. The carpenter gave consideration for the contractor's conditional promise to award the carpentry subcontract to him.

A park board in a large suburb announced that it was accepting bids for renovation work on its recreation center. A builder advertised for sub-bids for the electrical work, and a local electrician submitted to the builder by electronic bidding service a sub-bid of $130,000. However, due to the bidding service's negligence, the sub-bid that the builder received from the electrician read $30,000 instead of $130,000. Because this was the lowest sub-bid that the builder received for the electrical work, and $60,000 less than the next lowest sub-bid, the builder awarded the subcontract to the electrician. Based in part on the electrician's sub-bid, the builder came up with a bid for the job that beat out all of the competition and thus won the job. The electrician's best argument to successfully refuse to perform the resulting contract is: A. The contract would be unconscionable. B. The great difference between the $30,000 figure and the next lowest bid should have alerted the builder to the existence of a mistake in the sub-bid. C. The electrician was not responsible for the negligence of the bidding service. D. The builder's own negligence in not checking out all sub-bids precludes enforcement of the contract.

B. The great difference between the $30,000 figure and the next lowest bid should have alerted the builder to the existence of a mistake in the sub-bid.

Landholder was land-rich by inheritance but money-poor, having suffered severe losses on bad investments, but still owned several thousand acres of unencumbered timberland. He had a large family, and his normal, fixed personal expenses were high. Pressed for cash, he advertised a proposed sale of standing timber on a choice 2,000-acre tract. The only response was an offer by Logger, the owner of a large, integrated construction enterprise, after inspection of the advertised tract. Logger offered to buy, sever, and remove the standing timber from the advertised tract at a cash price of 70% lower than the regionally prevailing price for comparable timber rights. Landholder, by then in desperate financial straits and knowing little about timber values, signed and delivered to Logger a letter accepting the offer. If, before Logger commences performance, Landholder's investment fortunes suddenly improve and he wishes to get out of the timber deal with Logger, which of the following legal concepts affords his best prospect of effective cancellation? A. Undue Influence. B. Unconscionability. C. Physical Duress. D. Economic Duress.

B. Unconscionability. Not economic duress because Logger did not create the negative financial situation Landholder was in.

A consumer goes to a jewelry store and points out to the clerk a particular watch on display that he wishes to purchase. The clerk goes to the back of the store and emerges with a watch that he briefly shows to the consumer before wrapping it. The consumer buys the watch. When he gets home, the consumer discovers that the watch he purchased differs from the one that he pointed out in the display. Does the consumer have a claim for breach of warranty? A. Yes, because an express warranty was created by affirmation of fact or promise. B. Yes, because an express warranty was created by sample or model. C. No, because he had an opportunity to inspect the goods before purchase. D. No, because there was no language of warranty.

B. Yes, because an express warranty was created by sample or model.

In answer to a radio advertisement, a teenager two months shy of his 18th birthday contracted to buy a late model car from a car dealership. The agreement required a $1500 down payment with the remainder of the $7200 price to be paid in monthly installments to a local finance company. The teenager's first eight payments were made regularly until his driver's license was suspended. He then informed the company that no further payments would be forthcoming. The finance company sued for the remaining payments. The age of majority in the teenager's state is 18 years. Would the teenager be liable for the balance of the payments? A. Yes, because the car dealership was liable on the contract from the outset, notwithstanding the teenager's minority. B. Yes, because he kept the car for six months after reaching the age of majority. C. No, because he was a minor at the time of the contracting, and the contract is voidable by him. D. No, because he informed the finance company in a timely manner after his driver's license was suspended.

B. Yes, because he kept the car for six months after reaching the age of majority.

Buyer and Seller deal in electrical equipment. In its catalog, Buyer lists the prices for some of its products, but not for #4 conducting wire. On July 11, Buyer sends Seller this signed written message: "We wish immediately to have 4,000 feet of your #4 conducting wire. We do not see the price listed in your catalog, so we propose that the price be agreed on later. Please ship." On July 12, Seller responds with this signed writing: "Thank you. We will ship." Have Buyer and Seller formed a contract? A. Yes, because where the goods are bought and sold between merchants, price is not a material term. B. Yes, because notwithstanding the open price term, their interaction reflects finality of agreement. C. No, because parties do not form a contract if they agree that a material term will be subject to future agreement. D. No, because a seller who declines to publish its price shows an intention not to sell.

B. Yes, because notwithstanding the open price term, their interaction reflects finality of agreement.

On July 15, in a writing signed by both parties, Fixtures, Inc., agreed to deliver to Druggist on August 15 five storage cabinets from inventory for a total price of $5,000 to be paid on delivery. On August 1, the two parties orally agreed to postpone the delivery date to August 20. On August 20, Fixtures tendered the cabinets to Druggist, who refused to accept or pay for them on the ground that they were not tendered on August 15, even though they otherwise met the contract specifications. Assuming that all appropriate defenses are seasonably raised, will Fixtures succeed in an action against Druggist for breach of contract? A. Yes, because neither the July 15 agreement nor the August 1 agreement was required to be in writing. B. Yes, because the August 1 agreement operated as a waiver of the August 15 delivery term. C. No, because there was no consideration to support the August 1 agreement. D. No, because the parol evidence rule will prevent proof of the August 1 agreement.

B. Yes, because the August 1 agreement operated as a waiver of the August 15 delivery term.

An insurer offered a plan to cover an insured's catastrophic illnesses for the remainder of the insured's life in exchange for a large one-time payment at the inception of coverage. Because the program was experimental, the insurer would accept only a fixed number of applications during the enrollment period. A recent retiree in good health was one of the applicants accepted, and he enrolled in the program. He paid the one-time premium of $30,000 a few days before coverage began. The day after his coverage started, he was struck by a bus and killed. The executor of the retiree's estate reviewed the policy and immediately notified the bank to stop payment on it. The insurer then filed suit against the retiree's estate. Will the court compel the estate to pay the premium to the insurer? A. Yes, because the insurer necessarily declined to take another applicant during the enrollment period because of the retiree's promise to buy the policy. B. Yes, because the risk of the timing of the retiree's death was assumed by both parties and built into the cost of the contract. C. No, because the purpose of the contract between the retiree and the insurer had been frustrated. D. No, because it is unconscionable for the insurer to have charged the retiree so much for so little value received.

B. Yes, because the risk of the timing of the retiree's death was assumed by both parties and built into the cost of the contract.

A producer of cherries offers his crop to a fruit wholesaler. The producer shows the wholesaler sample cherries from his crop and, in a written agreement, the wholesaler agrees to buy the entire cherry crop for a certain price. When the cherries arrive, most of them are less plump and less ripe than the ones the producer had shown to the wholesaler. Does the wholesaler have a claim for breach of warranty? A. Yes, but only if the producer told the wholesaler the cherries would be plump and ripe. B. Yes, because the sample created an express warranty that the goods would conform to that sample. C. No, because no warranty was in writing. D. No, because the wholesaler bore the risk that the goods did not conform to the sample.

B. Yes, because the sample created an express warranty that the goods would conform to that sample.

A law professor sells a refrigerator that she owns to a student. After the purchase, Sears repossesses the refrigerator from the student because the law professor gave Sears a security interest in it. Assuming it was legal for Sears to do this, does the student have a claim against the professor for breach of warranty? A. Trick question! Article 2 doesn't apply to this transaction. B. Yes, the professor warranted that the goods were free from any security interest. C. No. Because the professor owned the refrigerator, she transferred good title to the student. D. No, because it is the responsibility of the buyer to check for security interests.

B. Yes, the professor warranted that the goods were free from any security interest.

Buyer, Inc. contracted in writing with Shareholder, who owned all of XYZ Corporation's outstanding stock, to purchase all of her stock at a specified price per share. At the time this contract was executed, Buyer's contracting officer said to Shareholder, "Of course, our commitment to buy is conditioned on our obtaining approval of the contract from Conglomerate, Ltd., our parent company." Shareholder replied, "Fine. No problem." Shareholder is willing and ready to consummate the sale of her stock to Buyer, but the latter refuses to perform on the ground (which is true) that Conglomerate has firmly refused to approve the contract. If Shareholder sues Buyer for breach of contract and seeks to exclude any evidence of the oral condition requiring Conglomerate's approval, the court will probably A. admit the evidence as proof of a collateral agreement. B. admit the evidence as proof of a condition to the existence of an enforceable obligation, and therefore not within the scope of the parol evidence rule. C. exclude the evidence on the basis of a finding that the parties' written agreement was a complete integration of their contract. D. exclude the evidence as contradicting the terms of the parties' written agreement, whether or not the writing was a complete integration of the contract.

B. admit the evidence as proof of a condition to the existence of an enforceable obligation, and therefore not within the scope of the parol evidence rule.

A seller agrees to sell a television set to a buyer for $1,000. After the deal was completed, the buyer discovers that other sellers in the area were selling the same television set for $750. The buyer claims that he is entitled to rescind the contract because the price must be reasonable. Is the buyer correct? A) Yes, because the purpose of the UCC is to ensure that prices are reasonable. B) Yes, because the market price is the reasonable price and this price is above market. C) No, because the reasonable price rule only applies when nothing is said as to price. D) No, because freedom of contract always allows a buyer to agree to pay an unreasonable price.

C) No, because the reasonable price rule only applies when nothing is said as to price.

During negotiations, the seller of a boat promised the buyer that the engine was warranted to be free of defects for a period of two years. The parties eventually signed a written agreement that contained no express warranty terms. Fourteen months after the purchase, the engine developed problems because of a defect. A court found that the writing is a complete and exclusive statement of the terms of agreement. Is the seller liable under the oral warranty? A) It remains to be determined as a question of fact whether the promise was made. B) Yes, because the oral promise does not contradict the terms of the writing. C) No, because the writing was a complete and exclusive expression of their agreement. D) No, because the buyer would have a claim even under the written warranty.

C) No, because the writing was a complete and exclusive expression of their agreement.

During negotiations, the seller of a boat promises the buyer that the engine was warranted to be free of defects for a period of two years. The parties then sign a written agreement that contains a warranty term with standard disclaimers of express and implied warranties, followed by language stating that "seller warrants that the engine will be free of defects for a period of one year." After 14 months, the engine develops problems because of a defect. A court finds that the writing is a partial integration and that the promise was actually made. Is the seller liable under the warranty? A) Yes, because the parties did not intend their writing to be final as to the warranty. B) Yes, because the oral promise does not contradict the terms of the writing. C) No, because the writing was a final expression of their agreement as to the warranty. D) No, because the buyer would have a claim even under the written warranty.

C) No, because the writing was a final expression of their agreement as to the warranty.

Buyer and Seller enter into a written contract whereby Buyer agrees to purchase Seller's house for $100,000 with closing on May 1. On April 30, Seller telephones Buyer and informs Buyer that he (Seller) will not be able to sell the house for $100,000 because the price of replacement homes has increased significantly in the past few weeks. Seller tells Buyer that he (Seller) will sell for $105,000. Buyer reluctantly accepts and the parties memorialize the modification in writing. At closing, after Seller signs the deed transferring the property to Buyer, Buyer tenders $100,000 but refuses to pay the additional $5,000. Is Buyer obligated to pay the additional $5,000? A. Yes, because the parties agreed in writing to the modification. B. Yes, if Seller was acting in good faith. C. No, because Seller gave no consideration for the modification. D. No, because Seller's $105,000 offer constituted a rejection.

C. No, because Seller gave no consideration for the modification.

On July 1, a cattle rancher offered to sell his ranch to a dairy farmer for $150,000. The dairy farmer paid the cattle rancher $1,000 to hold the offer open for a period of 30 days. On July 10, the dairy farmer wrote to the cattle rancher, telling him that he could not pay more than $100,000 for the ranch, and that if the cattle rancher would not agree to accept that amount, the dairy farmer would not go through with the deal. The dairy farmer received no reply from the cattle rancher. On July 29, the dairy farmer mailed a letter to the cattle rancher telling him that he accepted his offer to sell the ranch and enclosed a check for $150,000. The cattle rancher received this letter on August 1. Has a contract been formed between the parties for the sale of the ranch? A. No, because the dairy farmer's letter of July 10 terminated the cattle rancher's offer. B. No, because the cattle rancher did not accept the dairy farmer's counter-offer of $100,000. C. No, because the cattle rancher did not receive the dairy farmer's acceptance within 30 days. D. Yes, because the dairy farmer dispatched his acceptance of the cattle rancher's offer prior to the expiration of 30 days.

C. No, because the cattle rancher did not receive the dairy farmer's acceptance within 30 days.

A wholesaler persuaded a retailer to order a line of dolls for the Christmas season, even though the retailer was skeptical of the dolls' marketability. The contract stated that the retailer would "pay $1,500 for its order of 100 dolls provided that the dolls sold during the Christmas season." Some dolls did sell, but on February 12, the retailer had 80 of them in inventory. He sent the wholesaler notice that he would be returning the 80 dolls. The wholesaler replied that it did not want the dolls back, and that the retailer should continue to try to sell them. Despite this reply, the retailer sent the wholesaler a check for $300 and shipped the dolls to the wholesaler, who refused to accept them but did accept the check. Thereafter, the retailer held the dolls at his warehouse. The wholesaler brought an action to recover the $1,200 balance. The wholesaler will: A. Recover, because the retailer still has the dolls in his possession. B. Recover, because it was not a condition precedent that the dolls be sold during the Christmas season, but merely a convenient time for payment. C. Not recover, because sale during the Christmas season was a condition precedent to payment. D. Not recover, because accepting the $300 waived any rights that the wholesaler may have had to enforce the contract.

C. Not recover, because sale during the Christmas season was a condition precedent to payment.

A general contractor who wished to bid on a construction project solicited bids from a variety of subcontractors. Four electrical subcontractors submitted bids to the contractor in the amounts of $75,000, $85,000, $90,000, and $95,000, respectively. As he was making out his company's bid, which was higher than he wanted it to be, the contractor called the low bidder on the electrical work and told him, "We won't be able to do it with your present bid, but if you can shave off $5,000, I'm sure that the numbers will be there for us to get that project." The low bidder told the contractor that he could not lower his bid, adding that the bid he submitted was based on a $15,000 error, and he could not do the job for less than $90,000. The contractor lost the construction job and subsequently sued the low bidder. The low bidder is liable for: A. Breach of contract, because the mistake was not so unreasonably obvious as to make acceptance of his bid unconscionable. B. Breach of contract, because the mistake was unilateral. C. Nothing, because the low bidder rejected the contractor's counteroffer. D. Nothing, because even though the low bidder lacked authority to renege on its bid, the contractor suffered no damages because no bidder was willing to do the work for $70,000.

C. Nothing, because the low bidder rejected the contractor's counteroffer.

Breeder bought a two-month-old registered boar at auction from Pigstyle for $800. No express warranty was made. Fifteen months later, tests by experts proved conclusively that the boar had been born incurably sterile. If this had been known at the time of the sale, the boar would have been worth no more than $100. In an action by Breeder against Pigstyle to avoid the contract and recover the price paid, the parties stipulate that, as both were and had been aware, the minimum age at which the fertility of a boar can be determined is about 12 months. Which of the following will the court probably decide? A. Breeder wins, because the parties were mutually mistaken as to the boar's fertility when they made the agreement. B. Breeder wins, because Pigstyle impliedly warranted that the boar was fit for breeding. C. Pigstyle wins, because Breeder assumed the risk of the boar's sterility. D. Pigstyle wins, because any mistake involved was unilateral, not mutual.

C. Pigstyle wins, because Breeder assumed the risk of the boar's sterility.

A 17-year-old boy walked into a medical clinic and requested assistance with a deep cut he received when he fell off his bike. The doctor told the boy that if he agreed to work at the clinic for 45 hours a week for four weeks, he would stitch the wound and apply a medicated bandage. The clinic typically charges $225 for such treatment. Although it seemed like a lot of work for $225, the boy needed immediate medical attention, so he accepted the offer and promised to report for work the next day, after which the doctor treated the boy's injury. On his way home from the clinic, the medicated bandage fell into a ditch and was lost for good. The boy refused to report for work the next day. If the medical clinic sues the boy for $225 and loses, it will be because: A. The boy was a minor. B. The medicated bandage was destroyed and thus there was a breach of the implied warranty of merchantability. C. The contract was unconscionable. D. It was impossible for the boy to perform.

C. The contract was unconscionable.

A widget seller negotiates with a buyer for the purchase of 100 widgets for $10,000, with delivery 30 days from the signing of the agreement. After the parties sign the agreement, which contains a merger clause, the buyer asks the seller if delivery could be in 20 days and the seller says, "Yes, I promise we will do that." The seller does not deliver until 30 days from signing. The buyer sues the seller and the seller seeks to bar the evidence that the seller promised delivery in 20 days. Will the evidence be excluded under the parol evidence rule? A) Yes, because it contradicts a term of the written agreement. B) Yes, because it supplements a written agreement that is complete and exclusive. C) No, because it is offered for the purpose of interpreting the writing. D) No, because it is offered on an issue of modification.

D) No, because it is offered on an issue of modification.

A landowner advertised in the newspaper that he wished to sell 40 acres of land at $10,000 per acre. A rancher who was looking to expand his holdings was interested, so he came out to inspect the property. After the inspection, the rancher agreed to purchase the land for $400,000. A contract for the sale of the 40 acres was prepared and signed by the landowner and the rancher. The contract failed to state the purchase price. Later the rancher had a change of heart and refused to complete the purchase. In the landowner's lawsuit for breach of contract, the court should hold for: A) The landowner, because the parol evidence rule will not bar testimony that the rancher agreed to pay $400,000. B) The landowner, because the Statute of Frauds can be satisfied by combining the original advertisement and the written contract. C) The rancher, because the parol evidence rule will bar all evidence that he agreed to pay $400,000 for the land. D) The rancher, because the Statute of Frauds would require the contract to contain the price in order to be enforced.

D) The rancher, because the Statute of Frauds would require the contract to contain the price in order to be enforced.

Computers, Inc., contracted in writing with Bank to sell and deliver to Bank a mainframe computer using a new type of magnetic memory, then under development but not perfected by Computers, at a price substantially lower than that of a similar computer using current technology. The contract's delivery term was "F.O.B. Bank, on or before July 31." Assume that Computers tendered the computer to Bank on August 15, and that Bank rejected it because of the delay. If Computers sues Bank for breach of contract, which of the following facts, if proved, will best support a recovery by Computers? A. The delay did not materially harm Bank. B. Computers believed, on the assumption that Bank was getting a "super deal" for its money, that Bank would not reject because of the late tender of delivery. C. Computers' delay in tender was caused by a truckers' strike. D. A usage in the relevant trade allows computer sellers a 30-day leeway in the specified time of delivery, unless the usage is expressly negated by the contract.

D. A usage in the relevant trade allows computer sellers a 30-day leeway in the specified time of delivery, unless the usage is expressly negated by the contract.

Loomis, the owner and operator of a small business, encourages "wellness" on the part of his employees and supports various physical fitness programs to that end. Learning that one of his employees, Graceful, was a dedicated jogger, Loomis promised to pay her a special award of $100 if she could and would run one mile in less than six minutes on the following Saturday. Graceful thanked him, and did in fact run a mile in less than six minutes on the day specified. Shortly thereafter, however, Loomis discovered that for more than a year Graceful had been running at least one mile in less than six minutes every day as a part of her personal fitness program. He refused to pay the $100. In an action by Graceful against Loomis for breach of contract, which of the following best summarizes the probable decision of the court? A. Loomis wins, because there is a compelling inference that Loomis' promise did not induce Graceful to run the specified mile. B. Loomis wins, because Graceful's running of the specified mile was beneficial, not detrimental, to her in any event. C. Graceful wins, because running a mile in less than six minutes is a significantly demanding enterprise. D. Graceful wins, because she ran the specified mile as requested, and her motives for doing so are irrelevant.

D. Graceful wins, because she ran the specified mile as requested, and her motives for doing so are irrelevant.

Homeowner hired Incompetent, a professional carpenter, to construct a new deck on Homeowner's house. Under the arrangement, Incompetent was to provide all the materials (the wood, nails, etc.) and to complete construction for a price of $2,000. The materials themselves cost only $300. After Incompetent finished and received payment, Homeowner had a party. When Doug Drinker, one of the guests, walked out onto the deck, the deck collapsed and Drinker was severely injured. If Drinker sues Incompetent for breach of the UCC's implied warranty of merchantability and implied warranty of fitness for a particular purpose, Drinker most likely will: A. Prevail on both warranties. B. Lose on the implied warranty of merchantability because Incompetent is not a merchant, but prevail on the implied warranty of fitness for a particular purpose. C. Prevail on the implied warranty of merchantability because Incompetent is a merchant, but lose on the implied warranty of fitness for a particular purpose. D. Lose on both warranties.

D. Lose on both warranties. Because UCC warranties do not apply to services.

Jon and Margee work as coal miners. One day while working, Margee noticed that a dynamite charge was not properly placed and that Jon was at risk of being killed by the dynamite explosion. Margee rushed over to push Jon out of the way. In doing so, Margee absorbed some of the dynamite explosion and was crippled for the rest of her life. Jon was so moved by Margee's sacrifice that he promised to pay Margee $1,000 per month for the rest of Margee's life. Jon paid Margee the $1,000 per month for 5 years, but then Jon died. Jon's estate does not want to continue to pay Margee the $1,000 per month. Which of the following statements is most accurate? A. Margee will definitely not be allowed to enforce Jon's promise because it is supported only by past consideration. B. Margee might be allowed to enforce Jon's promise, but only if it was reduced to writing. C. Margee will definitely not be allowed to enforce Jon's promise because it was an offer to make a gift, which offer was automatically revoked upon Jon's death. D. Margee might be able to enforce the agreement if the court accepts the validity of the material benefit rule.

D. Margee might be able to enforce the agreement if the court accepts the validity of the material benefit rule.

Dumont, a real estate developer, was trying to purchase land on which he intended to build a large commercial development. Perkins, an elderly widow, had rejected all of Dumont's offers to buy her ancestral home, where she had lived all her life and which was located in the middle of Dumont's planned development. Finally, Dumont offered her $250,000. He told her that it was his last offer and that if she rejected it, state law authorized him to have her property condemned. Perkins then consulted her nephew, a law student, who researched the question and advised her that Dumont had no power of condemnation under state law. Perkins had been badly frightened by Dumont's threat, and was outraged when she learned that Dumont had lied to her. If Perkins asserts a claim based on misrepresentation against Dumont, will she prevail? A. Yes, if Dumont knew he had no legal power of condemnation. B. Yes, if Dumont tried to take unfair advantage of a gross difference between himself and Perkins in commercial knowledge and experience. C. No, if Dumont's offer of $250,000 equaled or exceeded the market value of Perkins's property. D. No, because Perkins suffered no pecuniary loss.

D. No, because Perkins suffered no pecuniary loss.

A burglar stole Collecta's impressionist painting valued at $400,000. Collecta, who had insured the painting for $300,000 with Artistic Insurance Co., promised to pay $25,000 to Snoop, a full time investigator for Artistic, if he effected the return of the painting to her in good condition. By company rules, Artistic permits its investigators to accept and retain rewards from policyholders for the recovery of insured property. Snoop, by long and skillful detective work, recovered the picture and returned it undamaged to Collecta. If Collecta refuses to pay Snoop anything, and he sues her for $25,000, what is the probable result under the prevailing modern rule? A. Collecta wins, because Snoop owed Artistic a preexisting duty to recover the picture if possible. B. Collecta wins, because Artistic, Snoop's employer, has a preexisting duty to return the recovered paining to Collecta. C. Snoop wins, because Collecta will benefit more from return of the $400,000 painting than from receiving the $300,000 policy proceeds. D. Snoop wins, because the preexisting duty rule does not apply if the promisee's (Snoop's) duty was owed to a third person.

D. Snoop wins, because the preexisting duty rule does not apply if the promisee's (Snoop's) duty was owed to a third person.

Bobby, age 17, enters into an oral agreement to purchase Sally's 1974 Ford Pinto for $500. Prior to either party performing, Sally repudiates. Bobby brings suit for breach of contract. If Sally wins, it will be because of: A. Bobby's age. B. The doctrine of duress. C. Undue influence. D. The Statute of Frauds.

D. The Statute of Frauds.

Buyer mailed a signed order to Seller that read: "Please ship us 10,000 widgets at your current price." Seller received the order on January 7 and that same day mailed to Buyer a properly stamped, addressed, and signed letter stating that the order was accepted at Seller's current price of $10 per widget. On January 8, before receipt of Seller's letter, Buyer telephoned Seller and said "I hereby revoke my order." Seller protested to no avail. Buyer received Seller's letter on January 9. Because of Buyer's January 8 telephone message, Seller never shipped the goods. Under the relevant and prevailing rules, is there a contract between Buyer and Seller as of January 10? A. No, because the order was an offer that could be accepted only by shipping the goods; and the offer was effectively revoked before shipment. B. No, because Buyer never effectively agreed to the $10 price term. C. Yes, because the order was, for a reasonable time, an irrevocable offer. D. Yes, because the order was an offer that Seller effectively accepted before Buyer attempted to revoke it.

D. Yes, because the order was an offer that Seller effectively accepted before Buyer attempted to revoke it.

John, a law student, is selling his car to Mary. He says, "This car will get 25 miles per gallon around town. Of course, I disclaim all express warranties." The parties then enter into a partially integrated agreement that makes no mention of warranties or warranty disclaimers. Has John given Mary a warranty that the car will get 25 miles per gallon around town? A. No, because John is not a merchant seller. B. No, because the statement is puffing. C. No, because all express warranties are disclaimed. D. Yes.

D. Yes.

A, an elderly and illiterate man, lives with and depends for his support on B, his nephew. B tells A that he will no longer support him unless A makes a contract to sell B a tract of land. A is thereby induced to make the proposed contract. Undue influence?

Even though B's conduct does not amount to duress, it amounts to undue influence because A is under the domination of B, and the contract is voidable by A.

A, who is not experienced in business, has for years been accustomed to rely in business matters on the advice of his friend, B, who is experienced in business. B constantly urges A to make a contract to sell to C, B's confederate, a tract of land at a price that is well below its fair value. A is thereby induced to make the contract. Undue influence?

Even though B's conduct does not amount to misrepresentation, it amounts to undue influence because A is justified in assuming that B will not act in a manner inconsistent with his welfare, and the contract is voidable.

Martha needed a new pair of shoes. She went to her local Skysheim shop and told the salesperson that she worked in the city's downtown area and had to walk eight blocks to get from her house to her commuter train and then six blocks from her train to her office. During the workday, she had to climb up and down stairs several times. She wanted shoes that were suitable for walking on concrete, had gripping power for stairs, and were comfortable. The salesperson went into his stockroom and brought out four different styles of Skysheim's 'Clouds,' reputably the most comfortable shoe on the market, designed for the type of use that Martha had in mind. Martha tried on each of the four pairs but did not like the way any of them looked. While walking around the store, however, Martha saw a shoe she did like--'Hobblers,' Skysheim's high-fashion shoe. She told the salesperson to bring her a pair to try. He did so and explained to Martha that "Hobblers were completely made of the finest leather and would last for years." Martha tried on the shoes and told the salesperson that she would take them. Martha bought the shoes and wore them twice. After wearing them only two times, the leather on both shoes began to tear apart, making them impossible to wear. She took the shoes back to the Skysheim shop and demanded her money back. Skysheim refused. If Martha sues to get her money back, under which theory(s) would she most likely prevail? I. Breach of the implied warranty of fitness for particular purpose. II. Breach of the implied warranty of merchantability. III. Breach of express warranty. IV. None of the above.

II. Breach of the implied warranty of merchantability. III. Breach of express warranty.

Restaurant averages 50 pies per month for the first six months. In month seven, Bakery opens two new locations and now demands 150 pies per month (50 pies for each location) from Bakery for $5 per pie. Is Bakery obligated to supply 150 pies?

No they can't, this quantity is unreasonable disproportionate to what they were doing in the past.

A, the owner of a newspaper, promises B that he will publish a statement about C known by A and B to be false and defamatory if B pays him $10,000. B pays A $10,000. Is the agreement enforceable?

No, A's promise is one to commit a tort and is unenforceable on grounds of public policy.

A pays B, an insurance company, $100,000 for an annuity contract under which B agrees to make quarterly payments to C, who is 50 years old, in a fixed amount for the rest of C's life. A and B believe that C is in good health and has a normal life expectancy, but C is afflicted with an incurable fatal disease and cannot live more than a year. Mutual mistake?

No, the contract is not voidable by A, because the court will allocate to A the risk of the mistake.

A contracts to sell and B to buy a tract of land. A and B both believe that A has good title, but neither has made a title search. The contract provides that A will convey only such title as he has, and A makes no representation with respect to title. In fact, A's title is defective. Mutual mistake?

No, the contract is not voidable by B, because the risk of the mistake is allocated to B by agreement of the parties.

A, seeking to induce B to make a contract to buy goods, tells B that he paid $10,000 for them. A knows that he paid only $8,000 for the goods. The facts being otherwise as stated in No. 3, A tells B only that the goods are worth $10,000. Misrepresentation?

No, the statement is one of opinion.

A, an unlicensed plumber, agrees to repair plumbing in B's home, for which B promises to pay A $1,000. A state statute, enacted to prevent the public from being victimized by incompetent plumbers and to protect the public health, requires persons doing plumbing to be licensed on the basis of an examination, the posting of a bond, and the payment of a fee, and makes violation a crime. A does the agreed work, but B fails to pay. Is the contract enforceable by A?

Probably not, because a court is likely to decide that B's promise is unenforceable on grounds of public policy.

A and B make an agreement for the sale of goods for $10,000, in which A promises to deliver the goods in his own truck at 6:00 a.m. on Sunday. A municipal parking ordinance makes unloading of a truck at that time and place an offense punishable by a fine of up to $50. A delivers the goods at the stated time and place, but B rejects them. Is the agreement enforceable?

Yes. Because the public policy manifested by the ordinance is not sufficiently substantial to outweigh the interest in the enforcement of B's promise, enforcement of his promise is not precluded on grounds of public policy.

Fruitko ordered from Orchard 500 bushels of No. 1 Royal Fuzz peaches at a specified price "for prompt shipment." Orchard promptly shipped No. 2 Royal Fuzz peaches instead of No. 1. The error in shipment was caused by the negligence of Orchard's shipping clerk. Do the parties have a contract and, if so, is Orchard in breach?

There is a contract and Orchard was in breach.

As it turns out, Restaurant is losing money purchasing the pies for $5. Restaurant asks Bakery to lower the price to $4, but Bakery refuses. As a result, Restaurant discontinues the sale of pies and instead starts selling cakes. Has Restaurant breached the contract it has with Bakery?

Yes. Loosing money isn't a good enough reason to shut it down, only if it's something outside their control (no one buys them, shortage on eggs, etc.)

Restaurant averages 50 pies per month for the first six months. In month seven, Bakery orders 55 pies because its business has improved. Is Bakery obligated to supply 55 pies?

Yes they are, 55 is required and close enough to what they normally ordered.

A, while negotiating with B for the sale of A's race horse, tells him that the horse has run a mile in a specified time. A is honestly mistaken, and, unknown to him, the horse has never come close to that time. B is induced by A's assertion to make a contract to buy the horse. Misrepresentation?

Yes, A's statement, although not fraudulent, is a material misrepresentation, and the contract is voidable by B.

Bakery and Restaurant agree in writing that Restaurant will buy all of the pies it requires for the next 12 months from Bakery at $5 per pie. (All ?s here dealing with UCC § 2-306) 1. Is this agreement legally enforceable?

Yes, requires is the keyword here, requirements contract, impose a duty of good faith.

A, an unlicensed plumber, agrees to repair plumbing in B's home, for which B promises to pay A $1,000. A state statute, enacted to prevent the public from being victimized by incompetent plumbers and to protect the public health, requires persons doing plumbing to be licensed on the basis of an examination, the posting of a bond, and the payment of a fee, and makes violation a crime. A does the agreed work, but B fails to pay. Is the contract enforceable by A? Same facts as 49, but assume A's work is defective. Is the contract enforceable by B?

Yes, since the ordinance was enacted to protect a class of persons to which B belongs against a class to which A belongs, enforcement of A's promise is not precluded on grounds of public policy and B can recover damages from A for breach of contract.

A, seeking to induce B to make a contract to buy his house, tells B that the plumbing is comprised of copper pipes. A does not know if the pipes are copper and, as it turns out, the pipes are not copper. B is induced by A's statement to make the contract. Misrepresentation?

Yes, the statement is a fraudulent misrepresentation, both because A does not have the confidence that he implies in its truth, and because he knows that he does not have the basis for it that he implies. The contract is voidable by B.

A, seeking to induce B to make a contract to buy goods, tells B that he paid $10,000 for them. A knows that he paid only $8,000 for the goods. Misrepresentation?

Yes, the statement is not one of opinion.

Would your answer change if Orchard's shipping clerk included a letter with the shipment of the peaches which stated: "We are out of No. 1 Royal Fuzz peaches, so we're shipping No. 2 peaches, which we hope will fill your needs. Please let us know."

Yes, this would become a counteroffer. Orchard would not be held in breach and neither would Fruitko if it denied this shipment.

Buyer, a motorcycle retailer, ordered one X750 motorcycle from Seller, a motorcycle manufacturer, for a stated price. Buyer's purchase order was silent regarding implied warranties. In reply, Seller faxed the following statement to Buyer: "We accept your offer to purchase one X750 motorcycle." On the second page of Seller's fax, Seller stated: "Seller hereby disclaims all implied warranties, including the implied warranty of merchantability." Same facts as prior slide, except assume that Seller's fax (page 1) also included the following statement: "This acceptance is expressly made conditional on Buyer's assent to all of the terms contained herein." What is the parties' contractual status when Buyer receives Seller's fax? a. The parties do not have a contract because Seller's fax effectively rejected Buyer's offer. b. The parties do not have a contract because Seller has yet to ship the motorcycle. c. The parties have a contract that contains the "disclaimer of warranties" clause. d. The parties have a contract that does not contain the "disclaimer of warranties" clause.

a. The parties do not have a contract because Seller's fax effectively rejected Buyer's offer.

Buyer, a motorcycle retailer, ordered one X750 motorcycle from Seller, a motorcycle manufacturer, for a stated price. Buyer's purchase order was silent regarding implied warranties. In reply, Seller faxed the following statement to Buyer: "We accept your offer to purchase one X750 motorcycle." On the second page of Seller's fax, Seller stated: "Seller hereby disclaims all implied warranties, including the implied warranty of merchantability." What is the parties' contractual status when Buyer receives Seller's fax? a. The parties do not have a contract because Seller's fax effectively rejected Buyer's offer. b. The parties do not have a contract because Seller has yet to ship the motorcycle. c. The parties have a contract that contains the "disclaimer of warranties" clause. d. The parties have a contract that does not contain the "disclaimer of warranties" clause.

d. The parties have a contract that does not contain the "disclaimer of warranties" clause.


Set pelajaran terkait

Pharm ch. 53 drugs for seizures and spasticity

View Set

Global and Cultural Effectiveness

View Set

Phases and outcome of the 30 Years War And corvee

View Set